Đến nội dung

Duy Thai2002 nội dung

Có 418 mục bởi Duy Thai2002 (Tìm giới hạn từ 30-03-2020)



Sắp theo                Sắp xếp  

#719425 $VMO2019$

Đã gửi bởi Duy Thai2002 on 13-01-2019 - 11:51 trong Thi HSG Quốc gia và Quốc tế

Nguồn: Facebook thầy Lữ
Mọi người vô chém ạ.

 

Các mem xem thử đề mới. Ai làm được thì vô chém nhé
Nguồn:the art of mathematics - trao đổi toán học

 

Tr2512:
Bài 1a: Theo định lý Rolle thì phương trình $f'=0$ tồn tại ít nhất 1 nghiệm thuộc $R$, đồng thời $f$ có tập xác định $(0;\infty)$ nên lim $\lim_{x\to - \infty}f' >0; \lim_{x\to -\infty}f' <0$ suy ra hàm số đạt GTLN trên R.

Hình gửi kèm

  • 49898377_2322032714497623_38128898587754496_n.jpg
  • 49848388_547403405728726_436004279363305472_n.jpg



#715773 ĐỀ THI CHỌN HSGQG TỈNH QUẢNG NGÃI

Đã gửi bởi Duy Thai2002 on 20-09-2018 - 17:57 trong Thi HSG cấp Tỉnh, Thành phố. Olympic 30-4. Đề thi và kiểm tra đội tuyển các cấp.

Kí hiệu $P(m,n)$ là thay $x$ bởi  $m$, $y$ bởi $n$ trong phương trình hàm đề bài.

$P(0,0)$ $f(-1)+(f(0))^{2}=-1$

$P(0,-1)$ $=> f(-1)+f(0)f(-1)=-1$

$=> f(0)(f(0)-f(-1))=0$ 

$<=> \begin{bmatrix}f(0)=0 & \\ f(0)=f(-1) & \end{bmatrix}$

Nếu $f(0)=f(-1)$ thì theo như trên suy ra được:

$f(0)^{2}+f(0)=-1$. Rõ ràng phương trình này vô nghiệm trên $\mathbb{R}$ nên loại TH này

$=> f(0)=0$. Theo $P(0,0)$ $=> f(-1)=-1$

$P(1,1)$ $=> (f(1))^{2}=1$

$<=> \begin{bmatrix}f(1)=1 & \\ f(1)=-1 & \end{bmatrix}$

Nếu $f(1)=1$ 

$P(x,1)$  

$=> f(x)+f(x-1)=2x-1$  (1)

Trong $(1)$ thay $x$ bởi $xy$:

$f(xy-1)+f(xy)=2xy-1$. 

$=> f(xy)=f(x)f(y)$

Trong (1), thay $x$ bởi $x^{2}+2x+1$:

$f(x^{2}+2x+1)+f(x^{2}+2x)=2x^{2}+4x+1$

$<=> f(x)f(x+2)+(f(x+1))^{2}=2x^{2}+4x+1$

Đặt $f(x+2)=a,f(x+1)=b,f(x)=c$

Ta có:

$\left\{\begin{matrix}a+b=2x+3 & & \\b+c=2x+1 & & \\ac+b^{2}=2x^{2}+4x+1 & & \end{matrix}\right.$( dễ dàng suy ra bằng việc thay đổi  giá trị $x$ trong (1))

$=> \left\{\begin{matrix}a=c+2 & & \\b=2x+1-c & & \\ac+b^{2}=2x^{2}+4x+1 & & \end{matrix}\right.$

$=> c(c+2)+(2x+1-c)^{2}=2x^{2}+4x+1$

Sau khi khai triển và thu gọn phương trình trên, ta được:

$2(c-x)^{2}=0$

$<=> c=x$

$<=> f(x)=x$ $\forall x\in \mathbb{R}$ . Thử lại thấy thỏa nên nhận hàm.

Nếu $f(1)=-1$

$P(x,1)$  

$=> f(x-1)-f(x) =2x-1$(2)

 Trong $(2)$ thay $x$ bởi $xy$:

$f(xy-1)+f(xy)=2xy-1$.

$=> f(xy)=-f(x)f(y)$

Trong (2), thay $x$ bởi $x^{2}+2x+1$:

$-f(x^{2}+2x+1)+f(x^{2}+2x)=2x^{2}+4x+1$

$<=>- f(x)f(x+2)+(f(x+1))^{2}=2x^{2}+4x+1$

Đặt $f(x+2)=a,f(x+1)=b,f(x)=c$

Ta có:

$\left\{\begin{matrix}b-a=2x+3 & & \\c-b=2x+1 & & \\b^{2}-ac=2x^{2}+4x+1 & & \end{matrix}\right.$( dễ dàng suy ra bằng việc thay đổi  giá trị $x$ trong (2))

$=> \left\{\begin{matrix}a=c-4x-4 & & \\b=c-2x-1& & \\b^{2}-ac=2x^{2}+4x+1 & & \end{matrix}\right.$

$=> (c-2x-1)^{2}-c(c-4x-4)=2x^{2}+4x+1$

Sau khi khai triển và thu gọn phương trình trên, ta được:

$2c=-2x^{2}$

$<=> c=-x^{2}$ $<=> f(x)=-x^{2}$ $\forall x\in \mathbb{R}$. 

Thử lại thấy thỏa nên nhận hàm trên.

Vậy $f(x)=x$, $f(x)=-x^{2}$ $\forall x\in \mathbb{R}$. 




#715406 ĐỀ THI CHỌN ĐỘI TUYỂN MÔN TOÁN NĂM HỌC 2018 - 2019 TRƯỜNG THPT CHUYÊN ĐHSP HÀ...

Đã gửi bởi Duy Thai2002 on 10-09-2018 - 22:13 trong Thi HSG cấp Tỉnh, Thành phố. Olympic 30-4. Đề thi và kiểm tra đội tuyển các cấp.

Bài 3. Ta có tính chất sau:

$a.b=(a,b)[a,b]$

Do đó , ta có bất biến là: tích hai số lúc đầu bằng tích hai số lúc sau.

Xét số $a_{1}$ mang giá trị  ban đầu là$a$ ( với $a_{1}$ là số ở ô thứ 1 và $a_{1}$  phải nhận giá trị là UCLN của hai số được thay)

Ta nhận nếu thay $a,b$ bởi $a',b'$   $a'=(a,b)$ thì $a_{1}< a,a_{1}<b$ . Cứ tiếp tục thay $a_{1}$ với  một số bất kì khác và lặp lại liên tục thì $a_{1}$ luôn giảm,.

Mà một số trong bảng thì lớn hơn hoặc bằng 1.Do đó, sẽ có lúc số $a_{1}$ phải bằng 1.  Tiếp tục giống như trên ở ô bất kì.

Thì sau hữu hạn bước, hoặc trên bảng có 2017 số 1 hoặc còn lại k số $1$ và 2018-k số, sao cho những số đó lập thành một cấp số nhân.

Do đó, quá trình kết thúc sau hữu hạn bước.




#715402 ĐỀ THI CHỌN ĐỘI TUYỂN MÔN TOÁN NĂM HỌC 2018 - 2019 TRƯỜNG THPT CHUYÊN ĐHSP HÀ...

Đã gửi bởi Duy Thai2002 on 10-09-2018 - 21:26 trong Thi HSG cấp Tỉnh, Thành phố. Olympic 30-4. Đề thi và kiểm tra đội tuyển các cấp.

2b). Ta có:

$x_{n}=a_{n}^{2}+b_{n}^{2}+c_{n}^{2}\geq \frac{(a_{n}+b_{n}+c_{n})^{2}}{3}=\frac{1}{3}$, $x_{n}=(a_{n}^{2}+b_{n}^{2}+c_{n}^{2})\leq (a_{n}+b_{n}+c_{n})^{2}=1$ ( do  $a_{n},b_{n},c_{n}$ đều dương.

$=> (x_{n})$ bị chặn.

Mặt khác, ta có: $x_{n+1}=\frac{3x_{n}^{2}-2x_{n}+1}{2}\leq x_{n}$

$<=> 3x_{n}^{2}-4x_{n}+1\leq 0$ ( đúng do $\frac{1}{3}\leq x_{n}\leq 1$)

$=> (x_{n})$ giảm và bị chặn nên $(x_{n})$ hội tụ. Gọi giới hạn dãy là $L$, $\frac{1}{3}\leq L\leq 1$

Chuyển qua giới hạn, ta có:

$L=\frac{3L^{2}-2L+1}{2}$

Giải pt trên được$\begin{bmatrix}L=\frac{1}{3} & \\ L=1 & \end{bmatrix}$

Kết luận.




#715398 ĐỀ THI CHỌN ĐỘI TUYỂN MÔN TOÁN NĂM HỌC 2018 - 2019 TRƯỜNG THPT CHUYÊN ĐHSP HÀ...

Đã gửi bởi Duy Thai2002 on 10-09-2018 - 21:09 trong Thi HSG cấp Tỉnh, Thành phố. Olympic 30-4. Đề thi và kiểm tra đội tuyển các cấp.

Bài 2 

a) Bằng quy nạp dễ dàng chứng minh $a_{n}+b_{n}+c_{n}=1$

Ta có: $x_{n}^{2}+\frac{(x_{n}-1)^{2}}{2}=(a_{n}^{2}+b_{n}^{2}+c_{n}^{2})^{2}+\frac{(-2a_{n}b_{n}-2b_{n}c_{n}-2c_{n}a_{n})^{2}}{2}=(a^{2}_{n}+b^{2}_{n}+c^{2}_{n})^{2}+2(a_{n}b_{n}+b_{n}c_{n}+c_{n}a_{n})^{2}$

Tới đây để ý một chút ta có thể phân tích:  $(a^{2}_{n}+b^{2}_{n}+c^{2}_{n})^{2}+2(a_{n}b_{n}+b_{n}c_{n}+c_{n}a_{n})^{2}=(a_{n}^{2}+2b_{n}c_{n})^{2}+(b_{n}^{2}+2c_{n}a_{n})^{2}+(c_{n}^{2}+2a_{n}b_{n})^{2}=a_{n+1}^{2}+b_{n+1}^{2}+c_{n+1}^{2}=x_{n+1}$

Do đó , $x_{n+1}=x_{n}^{2}+\frac{(x_{n}-1)^{2}}{2}$




#715392 ĐỀ THI CHỌN ĐỘI TUYỂN MÔN TOÁN NĂM HỌC 2018 - 2019 TRƯỜNG THPT CHUYÊN ĐHSP HÀ...

Đã gửi bởi Duy Thai2002 on 10-09-2018 - 20:48 trong Thi HSG cấp Tỉnh, Thành phố. Olympic 30-4. Đề thi và kiểm tra đội tuyển các cấp.

Câu 1. Đề anh đánh thiếu rồi. Phải là $f(f(x_{0}))$

Từ gt 

$=> x^{2}+ax+b=0$ có nghiệm kép là $f(x_{0})$

$<=> \Delta =0$

$<=> a^{2}=4b\geq 0$

$=> b\geq 0$

$=> f(x_{0})=\frac{-a}{2}$. Mà theo giả thuyết thì $f(x_{0})$ là nhiệm $f(x)$

$=> f(x_{0})=\frac{-a}{2}$

$<=> x_{0}^{2}+ax_{0}+\frac{a^{2}}{4}+\frac{a}{2}=0$

Mà $x_{0}$ là duy nhất nên pt $x^{2}+ax+\frac{a^{2}}{4} +a=0$

$<=> a^{2}-4(\frac{a^{2}}{4}+\frac{a}{2})=0$

$<=> a=0$.

Vậy $a,b$ là hai số thực  không âm.




#715272 Cho $n$ là số nguyên dương, xét tập hợp $S=\left \...

Đã gửi bởi Duy Thai2002 on 07-09-2018 - 06:26 trong Tổ hợp và rời rạc

Cách khác:

Gọi $a_{i},b_{i}$ là hai phần tử của nhóm thứ $i$ ,$1\leq i\leq 2n$

Ta có: $(\left | a_{i}-b_{i} \right |-1)^{2}\geq 0$

$=>a_{i}^{2}-2a_{i}b_{i}+b_{i}^{2}-2\left | a_{i}-b-{i} \right |+1\geq 0$

$=> a_{i}b_{i}\leq \frac{1}{2}(a_{i}^{2}+b_{i}^{2}-2\left | a_{i}-b_{i} \right |+1)\leq \frac{1}{2}(a_{i}^{2}+b_{i}^{2}-2+1)=\frac{1}{2}(a_{i}^{2}+b^{2}_{i}-1)$

$=> \sum_{i=1}^{2n}a_{i}b_{i}\leq \sum_{i=1}^{2n}\frac{1}{2}(a_{i}^{2}+b_{i}^{2}-1)=\frac{1}{2}\sum_{i=1}^{4n^{2}}(x^{2})-n$

ĐTXR $<=>$ phân thành các  tập  $(n,n+1)$




#715207 Chứng minh rằng $k\geqslant n.$

Đã gửi bởi Duy Thai2002 on 05-09-2018 - 15:45 trong Số học

Không biết bài trên tương tự bài 49 chỗ nào nhưng xin đưa ra một cách tiếp cận khác.

Không mất tính tổng quát, giả sử $n_{1}<n_{2}<n_{3}<...<n_{k}$

Giả sử $k< n$ 

*Giả sử $n_{k}< n$ $=>n_{k}\leq n-1$

Do $n_{k}>n_{k-1}$

$=> n_{k-1}\leq n-2$

Cứ lập luận như trên sẽ dẫn đến $n_{1}\leq n-k+1$ , $(n-k+1\geq 1)$

=>  $\sum_{m=1}^{k}2^{n_{m}}\leq 2^{n-1}+2^{n-2}+...+2^{n-k+1}$

Mặt khác , từ gt :

$=> \sum_{m=1}^{k}2^{n_{m}}\geq 2^{n}-1=2^{n-1}+2^{n-2}+...+2+1$

$=> 2^{n-1}+2^{n-2}+...+2^{n-k+1}\geq 2^{n-1}+2^{n-2}+...+2+1$ ( vô lí )

$=> n_{k}\geq n$

Nếu $n_{k-1}< n-1$ $<=> n_{k-1}\leq n-2$ thì theo như lập luận trên, $n_{k-2}\leq n-3$ và cứ lập luận tiếp tục như vậy sẽ dẫn tới $n_{1}\leq 0$

( vô lí theo gt)

Do đó, $n_{k-1}\geq n-1$, $n_{k-2}\geq n-2$,..., $n_{1}\geq 1$

Vậy phải có ít nhất $n$ số $n_{i}$, $i=\overline{1,k}$ thỏa yêu cầu bài toán hay $k\geq n$ (  vô lí theo điều giả sử)

Nên ta được $k\geq n$.




#715113 [TOPIC] Hai bài toán mỗi ngày.

Đã gửi bởi Duy Thai2002 on 03-09-2018 - 06:28 trong Chuyên đề toán THPT

Bài 47. Ta có: $(n+1)^{7}-n^{7}-1=7n(n+1)(n^{2}+n+1)^{2}$

Ta sẽ chứng minh có vô số số tự nhiên $n$ để $7n(n+1)$ luôn là số chính phương.

Điều này $<=>$ pt: $n(n+1)=7k^{2}$ có vô số nghiệm

Giả sử $k=x.y$ Ta xét trường hợp sau:

$\left\{\begin{matrix}n=7y^{2} & \\n+1=x^{2} & \end{matrix}\right.$

$=> x^{2}-7y^{2}=1$. Ta nhận thấy đây là phương trình Pell loại 1 nên có vô số nghiệm $x,y$ nguyên dương.

Do đó, ứng với mọi $x,y$ là nghiệm cuả phương trình Pell thì sẽ có $n$ tương ứng sao cho $7n(n+1)$  là số chính phương. Mà có vô số $x,y$ như vậy nên sẽ có vô số $n$.

Vậy ta có đpcm.




#715091 [TOPIC] Hai bài toán mỗi ngày.

Đã gửi bởi Duy Thai2002 on 02-09-2018 - 20:07 trong Chuyên đề toán THPT

bài 47 có 1 hướng tiếp cận là sử dụng phương trình pell. Lời giải sẽ gửi sau.



#714994 Một bài hình học phẳng trong đề thi Iran liên quan đến hàng điểm điều hòa

Đã gửi bởi Duy Thai2002 on 30-08-2018 - 22:48 trong Hình học phẳng

Định lí Blanchet mở rộng



#714831 [TOPIC] Hai bài toán mỗi ngày.

Đã gửi bởi Duy Thai2002 on 26-08-2018 - 16:43 trong Chuyên đề toán THPT

Lời giải khác bài 31.

Nhận thấy 7 là số nguyên tố, do đó 6 là cấp của 5 modulo 7

Ta có:$5^{n}\equiv -1\equiv 5^{3}$ (mod $7$)

$<=> n\equiv 3$ (mod $6$)

$=> n=6k+3$

Có: $v_{7}(5^{n}+1)=v_{7}(5^{6k+3}+1)=v_{7}(5^{3}+1)+v_{7}(2k+1)=1+v_{7}(2k+1)$

Theo giả thuyết, ta đươc5" 

$v_{7}(5^{n}+1)\geq 2000$

$<=> 1+v_{7}(2k+1)\geq 2000$

$<=> v_{7}(2k+1)\geq 1999$

$<=> 2k+1=7^{1999}m$

$=> n=3.7^{1999}m$




#711284 Chứng minh $2^{\frac{p-1}{2}} \e...

Đã gửi bởi Duy Thai2002 on 20-06-2018 - 11:09 trong Số học

Em muốn anh nêu ra mấy cái chứng minh đó luôn, cho nó dễ hiểu :D

Có trong sách Tài liệu chuyên toán giải tích 12 đó em. Em tham khảo trong đó




#711282 Chứng minh $2^{\frac{p-1}{2}} \e...

Đã gửi bởi Duy Thai2002 on 20-06-2018 - 11:06 trong Số học

Cái này là tính chất của thặng dư bình phương thôi.

Có:

$\left ( \frac{2}{p} \right )=(-1)^{n}$ ( Bổ đề Gauss)

$\left ( \frac{2}{p} \right )\equiv 2^{\frac{p-1}{2}}$ (mod p) ( Tiêu chuẩn Euler)

$=>2^{\frac{p-1}{2}}\equiv (-1)^{s}$ (mod p). (Đpcm)




#710669 $f(xf(x)+f(y))=f^{2}(x)+y$

Đã gửi bởi Duy Thai2002 on 11-06-2018 - 23:05 trong Hàm số - Đạo hàm

VP là hàm bậc nhất theo biến y nên có tập giá trị là $\mathbb{R}$. Do đó, VT có tập giá trị là  $\mathbb{R}$.

Suy ra $f$ toàn ánh.

$\Rightarrow$$\exists a$ sao cho:$f(a)=0$

Thay $x=a$, ta được:

$f(f(y))=y$

$=> f(f(x))=x$ $\forall x\in \mathbb{R}$

Thay $x$ bởi $f(x)$, ta được:

$f(xf(x)+f(y))=x^{2}+y$

Mà  $f(xf(x)+f(y))=f^{2}(x)+y$

Do đó:

$f^{2}(x)=x^{2}$

$<=> \begin{bmatrix} f(x)=x & \\f(x)=-x & \end{bmatrix}$

Ta sẽ chứng minh hai hàm thỏa với mọi x thuộc $ \mathbb{R}$. Giả sử tồn tại $c$ và $d$ sao cho $f(c)=c,f(d)=-d$

Thay $x$ bởi c và $y$ bởi d, ta được:

$f(c^{2}-d)=c^{2}+d$ ( vô lý vì $\begin{bmatrix}f(c^{2}-d)=c^{2}-d & \\f(c^{2}-d)=d-c^{2} & \end{bmatrix}$)

Từ đó dẫn tới hai hàm trên thỏa với mọi x thuộc $ \mathbb{R}$.

Vậy $f(x)=x,f(x)=-x$ $\forall x\in \mathbb{R}$




#710665 $x^{2}+y^{3}\geq x^{3}+y^{4...

Đã gửi bởi Duy Thai2002 on 11-06-2018 - 22:23 trong Bất đẳng thức và cực trị

Bài này có đăng trên diễn đàn rồi. Bạn chịu khó tìm.




#710413 $a^{2013}+b^{2013}=p^n$

Đã gửi bởi Duy Thai2002 on 09-06-2018 - 23:40 trong Số học

LTE




#710327 $x_n+\frac{x_{n-1}^2}{2}=\frac...

Đã gửi bởi Duy Thai2002 on 08-06-2018 - 21:39 trong Dãy số - Giới hạn

Câu 1. Chứng minh dễ dàng bằng quy nạp

Câu 2. Thì chuyển Công thức truy hồi qua giới hạn do dãy bị chặn nên có GHHH.




#710055 $\frac{ab}{a+b-c}+\frac{bc}...

Đã gửi bởi Duy Thai2002 on 05-06-2018 - 20:33 trong Bất đẳng thức và cực trị

Gợi ý sử dụng phép thế Ravi.




#709983 chứng minh $\frac{a_n+10}{2014}$ là số chí...

Đã gửi bởi Duy Thai2002 on 05-06-2018 - 10:53 trong Dãy số - Giới hạn

Giải pt sai pân tuyến tính bậc ba tìm cttq của $a_{n}$. Từ đó sẽ dễ dàng cm được yêu cầu của bài toán.




#709912 BR chia đôi EF

Đã gửi bởi Duy Thai2002 on 04-06-2018 - 14:37 trong Hình học

Hình vẽ sai rồi.




#709713 K là tâm nội tiếp tam giác AXY

Đã gửi bởi Duy Thai2002 on 01-06-2018 - 16:09 trong Hình học

Gọi $Z=AI\cap EF$

Ta thấy rằng $AK$ là phân giác của $\angle XAY$ nên ta cần chứng minh $YK$ là phân giác $\angle AYX$. Mà do $\angle AYX=\angle ACB$ nên nếu ta chứng minh được $\angle AYK=\angle ACI$ thì $\angle AYK=\angle ACI=\frac{1}{2}\angle ACB=\frac{1}{2}\angle AYX$  sẽ dẫn tới $YK$ là phân giác của $\angle XAY$, từ đó mà suy được $K$ là tâm đường tròn nội tiếp $\Delta AXY$

Ta sẽ đi chứng minh $\angle AYK=\angle ACI$. Thật vậy, ta có:

$\frac{AY}{AC}=cosA=2cos^{2}\frac{A}{2}-1=2\frac{AE^{2}}{AI^{2}}-1=\frac{2AE^{2}-AI^{2}}{AI^{2}}=\frac{AE^{2}-EI^{2}}{AI^{2}}=\frac{AZ.AI-IZ.AI}{AI^{2}}=\frac{AZ.AI-AI.ZK}{AI^{2}}=\frac{AI.AK}{AI^{2}}=\frac{AK}{AI}$

$=> \frac{AY}{AC }=\frac{AK}{AI}$

Lại có: $\angle YAK=\angle IAC=\frac{1}{2}\angle BAC$

$=> \Delta AYK \sim \Delta ACI$ ( c-g-c)

$=> \angle AYK=\angle ACI$

$=>$ Đpcm.




#709697 CMR $\frac{3}{4} < a,b,c <\frac...

Đã gửi bởi Duy Thai2002 on 01-06-2018 - 08:50 trong Bất đẳng thức và cực trị

Có:

$8=ab+bc+ca\leq b(5-b)+\frac{(a+c)^{2}}{4}=b(5-b)+\frac{(5-b)^{2}}{4}$

$=> \frac{3}{4}

Tương tự với a,c

$=>$ Đpcm.




#709595 $$\min\left \{ 2\,a-1/b,\, 2\,b-...

Đã gửi bởi Duy Thai2002 on 30-05-2018 - 14:39 trong Bất đẳng thức và cực trị

Bài này có một cách chứng minh rất hay là sử dụng phương pháp phản chứng. Lời giải như sau:

Nếu trong 3 số $a,b,c$ có hai số âm, một số dương giả sử $a>0,b>0$ và $c<0.$

Khi đó $2c-\frac{1}{a}< 0$. Do đó:  $\min\left \{ 2\,a-\frac{1}{b},\, 2\,b-\frac{1}{c},\,2\,c-\frac{1}{a}\right \}\leq 1$

Nếu cả 3 số đều dương:

Giả sử $\min\left \{ 2\,a-\frac{1}{b},\, 2\,b-\frac{1}{c},\,2\,c-\frac{1}{a}\right \}>  1 $

$=> 2a-\frac{1}{b}>1,2b-\frac{1}{c}>1,2c-\frac{1}{a}>1$

Có: $2a-\frac{1}{b}> 1$

$<=> 2ab> b+1$ . Tương tự, $2bc> c+1$, $2ca> a+1$. 

$=> 2\sum ab> a+b+c+3\geq 3\sqrt[3]{abc}+3=6$

$=> \sum ab> 3$

Lại có:

$2ab>b+1$ 

$=> 2> bc+c$. 

$=> 6> a+b+c+ab+bc+ca$

$<=> a+b+c< 6-ab-bc-ca< 6-3=3$ . (do $\sum ab> 3$)

$<=> a+b+c< 3$

Mặt khác, ta có:

$3(ab+bc+ca)\leq (a+b+c)^{2}< 3^{2}=9$

$<=> ab+bc+ca< 3$ ( vô lý vì ta vừa chứng minh ở trên là $ab+bc+ca> 3$)

Do đó:

$\min\left \{ 2\,a-\frac{1}{b},\, 2\,b-\frac{1}{c},\,2\,c-\frac{1}{a}\right \}\leq 1$

Kết hợp cả hai trường hợp lại , ta sẽ có đpcm.




#709520 CMR $FB \perp FD$.

Đã gửi bởi Duy Thai2002 on 29-05-2018 - 19:01 trong Hình học

Mô hình G7  IMO 2002 shortlist.